Você está na página 1de 7

AP/ADMS4503M

Assignment #1 Solutions
Winter 2012
Question 1 (6 marks)
A stock sells at $55 and is expected to pay a dividend of $1 per share at the end
of each quarter from today. The risk free interest rate is 4% per annum
continuously compounded for all maturities. An investor takes a long position on
the eleven-month forward contract today.
(a) What is the theoretical eleven-month forward price? (2 marks)
Answer
The present value (PV) of dividends, I, over the life of the forward contract is:
I = $1 e 0.040.25 + $1 e 0.040.5 + $1 e 0.040.75 = $2.940695 .

So the theoretical eleven-month forward price is:


F0 = (S 0 I) e

rT

= ($55 $2.940695) e

0.04

11
12

= $54.003573 .

(b) The current eleven-month forward price in the market is $53. Is there an
arbitrage opportunity? Please show all the details. (3 marks)
Answer
Since the market undervalues the forward contract, there is an arbitrage
opportunity to undertake. Specifically, we should short sell one share of the
stock, invest the proceeds, and take a long position on the forward contract. The
arbitrage profit to be realized is $1.003573. The details are illustrated in the
following table:

Short 1 share @ $55


1. Invest $0.990050 @ 4% for 3 months
2. Invest $0.980199 @ 4% for 6 months
3. Invest $0.970446 @ 4% for 9 months
4. Invest $52.059305 @ 4% for 11
months
Dividend
Long one forward contract @ $53
Profit

Today
$55.00
-$0.990050
-$0.980199
-$0.970446

3 months

6 months

9 months

$1.00
$1.00
$1.00

-$52.059305

$0.00

11 months

$54.003573
-$1.00

-$1.00

-$1.00

$0.00

$0.00

$0.00

-$53.00
$1.003573

AP/ADMS4503M

Assignment #1 Solutions Winter 2012

Alternatively, since the risk free interest rate is flat across maturities, we do not
need to divide the proceeds of our short sale and invest them for different
maturities, as done in the above table. Instead, we can invest $55 for eleven
months and pay the cumulative future value of dividends of $3.050521 in a lump
sum in eleven months and still realize the same arbitrage profit of $1.003573.
(c) Based on part b) above, what will be the value of the forward contract seven
months from now if the forward price in seven months is $51? (1 mark)
Answer
The delivery price, K, in the forward contract is equal to $53, the same as todays
forward price in the market. If in seven months from now, the forward price drops
to $51, then the value of the long forward contract is:
Vt = ( Ft K ) e

r ( T t )

= $(51 53) e

0.04

4
12

= $1.973510.

Question 2 (9 marks)
You are presented with the following market quotes:
Currency market
Spot exchange rate: CAD 1.6688 / GBP
Three-month forward: 1.6647
Six-month forward: 1.6612
Nine-month forward: 1.6585
Canadian Treasury bill yields
Three-month: 3.95%
Six-month: 4.15%
Nine-month: 4.30%

(a) Compute the three-month, six-month and nine-month interest rates that you
should expect to observe in the U.K. market. (3 marks)
Answer
Using the interest rate parity relationship, we have for the three-month forward
contract:

F0 = S0 e ( r rf )T = 1.6688 e ( 0.0395 rf )0.25 = 1.6647


rf = 4.9340%.
So the three-month U.K. interest rate should be 4.9340%. Likewise, we can find
that the six-month and nine-month U.K. interest rates should be 5.0629% and
5.1255%, respectively.

Page 2

AP/ADMS4503M

Assignment #1 Solutions Winter 2012

(b) Suppose that the six-month U.K. interest rate is 5.15% instead. Is there any
arbitrage profit to be made? Please show all the details. (3 marks)
Answer
If the six-month U.K. interest rate is 5.15%, the corresponding theoretical sixmonth forward exchange rate is
F0 = 1.6688 e ( 0.0415 0.0515 )0.5 = 1.6605,

which is overpriced by the market (1.6612 vs. 1.6605). We should borrow, say,
CAD 1,000, at the Canadian interest rate, convert it to GBP and invest in the U.K.
for six months, and take a short forward position to sell the GBP in six months
from now. The details are as follows:

Borrow CAD 1,000 @ 4.15% for six months


Convert to GBP @ spot CAD 1.6688 / GBP
Invest GBP in the U.K. @ 5.15% for six months
Short forward to sell GBP @ CAD 1.6612 / GBP in six
months
Profit

Today
CAD 1,000
GBP 599.2330
(GBP 599.2330)

CAD 0.00

In 6 months
(CAD 1,020.9668)
GBP 614.8636
CAD 1,021.4114
CAD 0.4446

(c) Now suppose that the nine-month U.K. interest rate is in fact 4.20%. Is there
an arbitrage opportunity? Please show all the details. (3 marks)
Answer
If the nine-month U.K. interest rate is 4.20%, the corresponding theoretical ninemonth forward exchange rate is
F0 = 1.6688 e ( 0.0430 0.0420 )0.75 = 1.6701,

which is undervalued by the market (1.6585 vs. 1.6701). We should borrow, say,
GBP 1,000, at the U.K. interest rate, convert it to CAD and invest in Canada for
nine months, and take a long forward position to buy the GBP in nine months
from today. The details are as follows:

Borrow GBP 1,000 @ 4.20% for nine months


Convert to CAD @ spot CAD 1.6688 / GBP
Invest CAD in Canada @ 4.30% for nine months
Long forward to buy GBP @ CAD 1.6585 / GBP in nine
months
Profit

Today
GBP 1,000
CAD 1,668.80
(CAD 1,668.80)

CAD 0.00

In 9 months
(GBP 1,032.0014)
CAD 1,723.4960
CAD 1,711.5743
CAD 11.9217

Question 3 (6 marks)

Page 3

AP/ADMS4503M

Assignment #1 Solutions Winter 2012

The following table shows some hypothetical data on the monthly changes in the
spot price and futures price of a commodity. Also assume that you want to
purchase 10,000 units of the commodity in one month from now.
S (in $)
F (in $)

4.00
-1.84

2.45
1.80

1.96
-2.00

1.87
1.50

2.36
3.00

-1.40
2.46

1.00
0.12

0.00
1.30

0.50
-2.00

-2.00
3.00

(a) If each futures contract is for a delivery of 100 units of the commodity, what
position should you take, i.e., long vs. short and number of contracts? (3 marks)
Answer
Following pages 57 58 in the textbook, we denote the ith observations on F
and S by xi and yi, respectively. The number of observations, n, is 10. From the
above table, we have:

= 7.3400, x i2 = 42.6316, y i = 10.7400, y i2 = 42.1206, x i y i = 7.3090.

So the estimate of F is:

2
i

n 1

( x i ) 2
n (n 1)

42.6316 (7.3400) 2

= 2.034263.
(10 1) 10 (10 1)

42.1206 (10.7400) 2

= 1.843482.
(10 1) 10 (10 1)

Likewise, the estimate of S is:

2
i

n 1

( y i ) 2

n (n 1)

The estimate of is:

n x i yi x i yi

[n x i2 ( x i ) 2 ][n y i2 ( y i ) 2 ]
=

10 (7.3090) (7.3400 10.7400)


[10 42.6316 (7.3400) 2 ] [10 42.1206 (10.7400) 2 ]

= 0.450123.

The minimum variance hedge ratio, h*, is:

h* =

S
1.843482
= 0.450123
= 0.407909.
F
2.034263

The number of futures contracts required, N*, is:

Page 4

AP/ADMS4503M

Assignment #1 Solutions Winter 2012

h * Q A 0.407909 10,000
N =
=
41.
QF
100
*

Since the hedge ratio is negative, we should take a short position on 41 futures
contracts.
(b) The futures price today is $47 per unit. Suppose in one month from today
when you close out your hedging position the futures price and the spot price at
that time are both $53 per unit, what is the effective cost per unit for the
commodity? Did the hedge work in your favour? (3 marks)
Answer
The price paid for 10,000 units of commodity in the spot market in one month
from now is $530,000 (= $53 10,000). The gain/loss from the short futures
position is -$24,600 (= $[47 53] 41 100). So the effective cost per unit for
the commodity is $55.46 (= $554,600 / 10,000). The hedge did not work in your
favor because the futures price has risen from $47 to $53 over the holding period
and you have taken a short futures position.
Question 4 (6 marks)
A portfolio manager wants to use the CME futures contracts on the S&P 500
index to hedge her portfolio worth $50 million over the next six months. The beta
of the portfolio is 1.5. The index level is now 1,120 and each contract is on $250
times the index. The S&P 500 dividend yield is 2% and the risk free interest rate
is 4%. Both rates are annual and continuously compounded.
(a) What is the six-month futures price? (1 mark)
Answer
F0 = S 0 e ( r q )T = 1,120 e ( 0.04 0.02 )0.5 = 1,131.2562.

(b) How many long or short futures contracts should the manager take? (2
marks)
Answer
Since the manager is long on her portfolio, she should take a short position on
the futures contracts in order to hedge. Specifically, the number of futures
contracts that she should short is:

N* =

P
$50,000,000
= 1.5
= 265.1919 265.
F
$250 1,131.2562

Page 5

AP/ADMS4503M

Assignment #1 Solutions Winter 2012

(c) What would be the net gain/loss in four months from today in each of the
following scenarios if the manager sells off her portfolio and closes out her
futures position at that time? (3 marks)
Portfolio value
in four months
$69 million
$43 million

Scenario
1
2

S&P 500 futures


in four months
1,320
990

Answer
The net gain/loss in each scenario is the sum of the gain/loss on her portfolio and
the gain/loss on her short futures position. In particular,
in Scenario 1: $(69,000,000 50,000,000) + 265 $250 (1,131.2562 1,320)
= $6,495,723.25;
in Scenario 2: $(43,000,000 50,000,000) + 265 $250 (1,131.2562 990) =
$2,358,223.25.
Question 5 (6 marks)
A stock is expected to pay a dividend of $0.50 per share in two, five and eight
months from now, respectively. The spot stock price is $60 and the risk free
interest rate is 3% per annum continuously compounded. Today an investor
takes a short position on a ten-month forward contract on the stock.
(a) What are the forward price today and the initial value of the forward contract?
(3 marks)
Answer
The PV of dividends, I, over the life of the forward contract is:
I = $0.50 e

0.03

2
12

+ $0.50 e

0.03

5
12

+ $0.50 e

8
0.03
12

= $1.481394 .

So the theoretical ten-month forward price is:


F0 = (S 0 I) e

rT

= ($60 $1.481394) e

10
0.03
12

= $60.000012.

The initial value of the forward contract is zero by the contract design. The
delivery price, K, of the forward contract is also $60.000012.
(b) Suppose seven months later, the stock price drops to $52, and the investor
decides to close out her short position. What is the price at which she should sell
Page 6

AP/ADMS4503M

Assignment #1 Solutions Winter 2012

her position? (3 marks)


Answer
Seven months later, the PV of the remaining dividend, I, is:
I = $0.50 e

0.03

1
12

= $0.498752 .

The forward price (for a remaining maturity of three months) at that time is
given by:
F0 = (S 0 I) e

rT

= ($52 $0.498752) e

3
0.03
12

= $51.888959.

The price at which she should sell her forward position is the value of the
forward contract at that time, which is:
Vt = (K Ft ) e r( T t ) = $(60.000012 51.888959) e

0.03

3
12

= $8.050448.

Page 7

Você também pode gostar